You are on page 1of 58

th

30
Iranian
Mathematical
Olympiad

2012-2013
Young Scholars Club
Ministry of Education, I.R.Iran
www.ysc.ac.ir

30th
Iranian Mathematical Olympiad

Selected Problems and Their Solutions

30th Iranian

Mathematical Olympiad

Selected Problems and Their Solutions

This booklet is prepared by Goodarz Mehr, Hesam Rajabzadeh and


Morteza Saghaan.
With special thanks to Ali Khezeli, Atieh Khoshnevis, Mahan Malihi, Omid
Naghshineh Arjmand, Sam Nariman and Erfan Salavati.

c Young Scholars Club 2012-2013. All rights reserved.


Copyright
Ministry of education, Islamic Republic of Iran.
www.ysc.ac.ir
3

Iranian Team Members in the 54th IMO (Santa Marta-Colombia)


From left to right:

Tina Torkaman
Aref Sadeghi
Alek Bedroya
Seyyed Mohammadhossein Seyyedsalehi
Mohammad Javad Shabani Zahraei
Mahan Tajrobehkar

Contents
Preface..................................................................................................................... 9
Problems

Second Round......................................................................................................... 13
Third Round........................................................................................................... 14
Team Selection Test................................................................................................ 16
Solutions

Second Round........................................................................................................ 21
Third Round.......................................................................................................... 26
Team Selection Test............................................................................................... 36

Preface
The 30th Iranian National Mathematical Olympiad consisted of four rounds. The
rst round was held on 24 February 2012 all over the country. The exam consisted of 10
short-answer questions and 15 multiple-choice questions to be solved in 3 hours. In total
more than 25000 students participated in the exam and 1446 of them were validated for
the next round.
The second round was held on 30 April and 1 May 2012. In each day, participants
were given 3 problems to be solved in 4.5 hours. In this round, from a total of 1446
participants, 43 of them were chosen to participate in the third round.
The examination of the third round consisted of ve separate exams, and a nal exam
containing 8 questions, each having it's specied time to be solved in. In the end, 10 people were awarded Bronze Medal, 20 people were awarded Silver Medal, and 13 people were
awarded Gold Medal, which the following list represents the names of the Gold Medalists:
Jalal Ahmadi
Mohammad Javad Baferooni
Alek Bedroya
Hossein Hazrati
Mohammad Amin Ketabchi
Hadi Khodabande
Aref Sadeghi
Seyyed Mohammadhossein Seyyedsalehi
Mohammad Javad Shabani Zahraei
Mahan Tajrobehkar
Seyyed Alireza Tavakoli
Tina Torkaman
Mobin Yahyazade Jelodar
The Team Selection Test was held on 6 days, each day having 3 problems to be solved
in 4.5 hours. In the end, the top 6 participants were chosen to participate in the 54th
IMO as members of the Iranian Team.
In this booklet, we present 6 problems of the Second Round, 8 problems of the nal
exam of the Third Round, and 18 problems of the Team Selection Test, together with
their solutions.
It's a pleasure for the authors to oer their grateful appreciation to all the people
who have contributed to the conduction of the 30th Iranian Mathematical Olympiad, including the National Committee of Mathematics Olympiad, problem proposals, problem
selection teams, exam preparation teams, coordinators, editors, instructors and all who
have shared their knowledge and eort to increase the Mathematics enthusiasm in our
country, and assisted in various ways to the conduction of this scientic event.

Problems
11

Second Round
1. (Morteza Adl) Consider a circle C1 and a point O on it. Circle C2 with center O,
intersects C1 in two points P and Q. C3 is a circle which is externally tangent to C2 at
R and internally tangent to C1 at S and suppose that RS passes through Q. Suppose X
and Y are second intersection points of P R and OR with C1 . Prove that QX is parallel
to SY .
( p.21)
2. (Morteza Saghaan) Suppose n is a natural number. In how many ways can we
place numbers 1, 2, ...., n around a circle such that each number is a divisor of the sum
of it's two adjacent numbers?
( p.21)
3. (Mahyar Sedgaran, Amin Nejatbakhsh) Prove that if t is a natural number then
there exists a natural number n > 1 such that (n, t) = 1 and none of the numbers
n + t, n2 + t, n3 + t, .... are perfect powers.
( p.23)
4. (Morteza Saghaan) a) Do there exist 2-element subsets A1 , A2 , A3 , ... of natural
numbers such that each natural number appears in exactly one of these sets and also for
each natural number n, sum of the elements of An equals 1391 + n?
b) Do there exist 2-element subsets A1 , A2 , A3 , ... of natural numbers such that each
natural number appears in exactly one of these sets and also for each natural number n,
sum of the elements of An equals 1391 + n2 ?
( p.23)
5. (Sahand Seifnashri) Consider the second degree polynomial x2 + ax + b with
real coecients. We know that the necessary and sucient condition for this polynomial to have roots in real numbers is that its discriminant, a2 4b, be greater than or
equal to zero. Note that the discriminant is also a polynomial with variables a and b.
Prove that the same story is not true for polynomials of degree 4: Prove that there does
not exist a 4 variable polynomial P (a, b, c, d) such that the fourth degree polynomial
x4 + ax3 + bx2 + cx + d can be written as the product of four 1st degree polynomials if
and only if P (a, b, c, d) 0. (All the coecients are real numbers.)
( p.24)
6. (Mehdi E'tesami Fard) The incircle of triangle ABC , is tangent to sides BC, CA
and AB at D, E and F respectively. The reection of F with respect to B and the
reection of E with respect to C are T and S respectively. Prove that the incenter of
triangle AST is inside or on the incircle of triangle ABC .
( p.24)

13

Third Round
1. (Morteza Saghaan) Let G be a simple undirected graph with vertices v1 , v2 , ..., vn .
We denote the number of acyclic orientations of G with f (G).
a) Prove that f (G) f (G v1 ) + f (G v2 ) + + f (G vn ).
b) Let e be an edge of the graph G. Denote by G the graph obtained by omitting e and
making it's two endpoints as one vertex. Prove that f (G) = f (G e) + f (G ).
c) Prove that for each > 1, there exists a graph G and an edge e of it such that
f (G)
90 minutes ( p.26)
f (Ge) < .
2. (Ali Khezeli) Suppose S is a convex gure in plane with area 10. Consider a chord
of length 3 in S and let A and B be two points on this chord which divide it into three
equal parts. For a variable point X in S {A, B}, let A and B be the intersection
points of rays AX and BX with the boundary of S . Let S be those points of X for
which AA > 31 BB . Prove that the area of S is at least 6.
105 minutes ( p.26)
3. (Amirhossein Gorzi) Prove that for each n N there exist natural numbers
a1 < a2 < < an such that (a1 ) > (a2 ) > > (an ).
50 minutes ( p.28)
4. (Hamidreza Ziarati) We have n bags each having 100 coins. All of the bags have
10 gram coins except one of them which has 9 gram coins. We have a balance which can
show weights of things that have weight of at most 1 kilogram. At least how many times
shall we use the balance in order to nd the dierent bag?
75 minutes ( p.28)
5. (Mostafa Einollahzade, Erfan Salavati) We call the three variable polynomial P
cyclic if P (x, y, z) = P (y, z, x). Prove that cyclic three variable polynomials P1 , P2 , P3
and P4 exist such that for each cyclic three variable polynomial P , there exists a four
variable polynomial Q such that

P (x, y, z) = Q(P1 (x, y, z), P2 (x, y, z), P3 (x, y, z), P4 (x, y, z)).
60 minutes ( p.29)
6. (Mostafa Einollahzade) a) Prove that a real number a > 0 exists such that for each
natural number n, there exists a convex n-gon P in plane with lattice points as vertices
such that the area of P is less than an3 .
b) Prove that there exists b > 0 such that for each natural number n and each n-gon P
in plane with lattice points as vertices, the area of P is not less than bn2 .
c) Prove that there exist , c > 0 such that for each natural number n and each n-gon
P in plane with lattice points as vertices, the area of P is not less than cn2+ .
105 minutes ( p.30)
14

7. (Erfan Salavati) The city of Bridge Village has some highways. Highways are
closed curves that have intersections with each other or themselves in 4-way crossroads.
Mr.Bridge Lover, mayor of the city, wants to build a bridge on each crossroad in order
to decrease the number of accidents. He wants to build the bridges in such a way that in
each highway, cars pass above and under a bridge alternatively. By knowing the number
of highways determine whether this action is possible or not.
50 minutes ( p.33)

8. (Morteza Saghaan) a) Does there exist an innite subset S of the natural numbers, such that S = N, and such that for each natural number n S , exactly n members
of S are coprime with n?
b) Does there exist an innite subset S of the natural numbers, such that for each natural
number n S , exactly n members of S are coprime with n?
75 minutes ( p.34)

15

Team Selection Test


1. (Mehdi E'tesami Fard) In acute triangle ABC , let H be the foot of the perpendicular from A to BC and also let J and I be the B -excenter of triangle ABH and
C -excenter of triangle ACH , respectively. If P is the point of tangency of the incenter
of triangle ABC with side BC , prove that points I, J, P and H lie on the same circle.
(p.36)
2. (Ali Khezeli) At most how many subsets of the set {1, 2, ..., n} can be selected in
such a way that if A and B are two selected subsets and A B , then |B A| 3? (By
|X| we mean number of elements of the set X .)
(p.37)
3. (Morteza Saghaan) For nonnegative integers m and n, the sequence a(m, n) of
real numbers is dened as follows: a(0, 0) is equal to 2, and for each natural number n,
a(0, n) = 1 and a(n, 0) = 2. Also for m, n N:

a(m, n) = a(m 1, n) + a(m, n 1)


Prove that for each natural number k , all roots of the polynomial Pk (x) =
1 2i)xi are real numbers.

i=0 a(i, 2k +

(p.37)

4. (Shayan Dashmiz) Suppose m and n are two nonnegative integers. In the Philosopher's Chess, the chessboard is an innite array of same regular hexagon cells.
The Phoenix piece, which is a special
piece in this kind of chess, moves as follows:
At rst, the Phoenix selects one of
the six directions and moves m cells in
that direction.
Then it turns 60 degrees clockwise and moves n cells in
that new direction to get to the nal
point.
At most how many cells of the Philosopher's Chess' chessboard exist in such a way
that one cannot start from one of them and
reach another one with a nite number of
movements of the Phoenix piece?

(p.38)

5. (Mahan Malihi) Do there exist natural numbers a, b and c such that a2 + b2 + c2


is divisible by 2013(ab + bc + ca)?
(p.39)
6. (Ali Khezeli) Let X, Y, X and Y be four points on a line l with the same order as
written. Consider arcs 1 and 2 with end points {X, Y } and arcs 1 and 2 with end
16

points {X , Y } on one side of l such that 1 is tangent to 1 and 2 is tangent to 2 .


Prove that the external common tangent of 1 and 2 and the external common tangent
of 1 and 2 meet on l.
(p.41)
7. (Mohammad Ali Karami) Nonnegative real numbers p1 , p2 , ..., pn and q1 , q2 , ..., qn
are given such that

p1 + p2 + ... + pn = q1 + q2 + ... + qn .
Among all matrices with nonnegative real entries for which sum of entries of the ith row
is pi and sum of entries of the j th column is qj , nd the maximum value that the trace
of the matrix can have.
(p.44)
8. (Yahya Motevassel) Find all innite arithmetic progressions a1 , a2 , ... for which
there exists natural number N > 1 such that for each k N:

a1 a2 ak |aN +1 aN +2 aN +k .
(p.46)
9. (Mohammad Jafari) Find all functions f, g : R+ R+ such that f is an increasing
function and also for each x, y R+ :

f (f (x) + 2g(x) + 3f (y)) = g(x) + 2f (x) + 3g(y)


g(f (x) + y + g(y)) = 2x g(x) + f (y) + y.
(p.46)
10. (Morteza Saghaan) To each edge of a graph, we have assigned a real number in
a way that for each even walk of this graph, sum of the numbers on edges of the walk,
with minus and plus signs assigned alternatively, is zero. Prove that we can assign a
real number to each vertex so that for each edge, the number assigned to it is the sum
of numbers assigned to it's endpoints. (A walk is a closed path of edges that can have
(p.47)
repeated number of vertices or edges.)
11. (Amirhossein Gorzi) Suppose we have a triangle with side lengths a, b and c such
that a b c. Prove that

a(a + b ab) + b(a + c ac) + c(b + c bc) a + b + c.


(p.48)
12. (Ali Zamani) Quadrilateral ABCD is inscribed in circle . Suppose that I1 and
I2 are incenters of triangles ACD and ABC and r1 and r2 are inradiuses of triangles
ACD and ABC , respectively. We know that r1 = r2 . Circle is tangent to sides AB
and AD and also is tangent to circle at T . Tangents at T and A to circle intersect
17

each other in K . Prove that I1 , I2 and K are collinear.

(p.48)

13. (Ali Zamani) Point P is an arbitrary point inside acute triangle ABC and A1 , B1
and C1 are reections of P with respect to sides of triangle ABC . Prove that the centroid
of triangle A1 B1 C1 is inside triangle ABC .
(p.50)
14. (Amirhossein Gorzi) We have drawn n rectangles in plane
where n is a natural
number. Prove that among 4n resulting right angles, at least 4n distinct right angles
exist.
(p.50)
15. (Amirhossein Gorzi) a) Does there exist a sequence of natural numbers a1 < a2 <
for which there exists a natural number N in a way that for each natural number
m N , am is divisible by exactly d(m) 1 terms of the sequence? (d(n) is the number
of divisors of natural number n.)
b) Does there exist a sequence of natural numbers a1 < a2 < for which there
exists a natural number N in a way that for each natural number m N , am is divisible
by exactly d(m) + 1 terms of the sequence?
(p.50)
16. (Amirhossein Gorzi) The function f : Z Z has the property that for all integers
m and n

f (m) + f (n) + f (f (m2 + n2 )) = 1.


We know that integers a and b exist such that f (a) f (b) = 3. Prove that integers c
and d can be found such that f (c) f (d) = 1.
(p.51)
17. (Mahan Malihi) AD and AH are angle bisector and altitude of vertex A, respectively. The perpendicular bisector of AD, intersects semicircles with diameters AB and
AC which are drawn outside triangle ABC in X and Y , respectively. Prove that the
quadrilateral XY DH is concyclic.
(p.51)
18. (Ali Khezeli) A special kind of parallelogram tile is made up by attaching the
legs of two right isosceles triangles of side length 1. We want to put a number of these
tiles on the oor of an n n room such that the distance from each vertex of each tile
to the sides of the room is an integer and also no two tiles overlap. Prove that at least
an area n of the room will not be covered by the tiles.
(p.52)

18

Solutions
19

Second Round
1

. Let l be the common tangent of C1 and C3 . Then:

SR = 2RSl =

2QSl

SR =

RQ

SQ

= 2RP Q = XQ

SQ = XQ SQ = XQ. (1)

So it is sucient to prove that QX = XY . Notice that O is the center of C2 , so:

OP = OR OP R = ORP OX = OP + XY

OQ + QX = OP + XY QX = XY . (2)
(1), (2) SQ = XQ = XY SY QX.

2 . We call an arrangement good if it satises the problem requirements. Consider one


of these arrangements. n should be a divisor of sum of it's two adjacent numbers, but the
sum of these two numbers is less than n + n = 2n. So sum of the two adjacent numbers
of n must be n. Thus for a natural number k , 1 k n 1, these two numbers are k
and n k . As a result, we see that by omitting n, the condition of the problem is still
satised, because n n k (mod k) and n k (mod n k), which means k and n k
still divide sum of their two adjacent numbers. So by omitting n, we reach to a good
arrangement of numbers 1, 2, ..., n 1. For n = 1 and n = 2, the answer to the question
is 1; and for n = 3 the answer is 2. Now, by using induction, we prove that the answer
21

to the question is 4 for odd natural numbers n greater than 3, and 2 for even natural
numbers n greater than 2.
Base case of the induction: n = 4

Suppose that our assumption holds for n = k . A good arrangement of numbers


1, 2, ..., k + 1 will become a good arrangement of numbers 1, 2, ..., k by omitting k + 1.
If k+1 is odd, by the induction hypothesis good arrangements of numbers 1, 2, ..., k are

k
2

and k + 1 could only be placed between numbers 1 and k or between numbers k2 and
+ 1 which have a sum of k + 1. Thus we have 4 good arrangements in this case.

are

If k + 1 is even, by the induction hypothesis good arrangements of numbers 1, 2, ..., k


22

and k + 1 could only be placed between the two numbers 1 and k in the two bottommost arrangements. So we have 2 good arrangements in this case.
In conclusion, The answer to the problem for n = 1 and n = 2 is 1, for n = 3 and for
all other even numbers is 2, and for all other odd numbers is 4.

3 . Let p be a prime factor of t + 1 and let pm ||t + 1. Dene K = m!(pm+1 ) and


n = (t(t + 1) + 1)K . Now, we have nr + t 1 + t (mod pm+1 ), so if nr + t = bs then
s m. On the other hand we have
(
)s
K
r
Kr
r
s
n = (t(t + 1) + 1) = (t(t + 1) + 1)
= n0 s ,
(Because s|m!|K)
so

t = bs nr = bs n0 s (n0 + 1)s n0 s sn0 2n0 > t


which is a contradiction.

. a) Let Ai = {xi , yi }. So xi + yi = 1391 + i, but:


13912 + 1391 696 =

1391

(xi + yi )

i=1

= x1 + x2 + + x1391 + y1 + y2 + + y1391
1 + 2 + + 2 1391 = 1391 2783.
23

Contradiction!
b) We construct such Ai 's inductively. Let A1 = {1, 1391}. Now, assume that
A1 , A2 , . . . and Ak are constructed. Let xk+1 be the least positive integer which is not in
any of Ai 's (1 i k). Obviously xk+1 2k + 1, so the number

yk+1 = 1391 + (k + 1)2 xk+1 1391 + k 2


is not in any of Ai 's too (because sum of the elements of each Ai , (1 i k) is at
most 1391 + k 2 until now). So we can have Ak+1 = {xk+1 , yk+1 }, and in these Ai 's, each
positive integer has appeared exactly once (because of the method of choosing xk+1 in
each step).

5 . If we put a = c = 0, polynomial x4 + bx2 + d can be written as product of four


linear terms if and only if quadratic polynomial y 2 + by + d has two nonnegative roots.
Therefore P (0, b, 0, d) 0 if and only if b 0, d 0 and b2 4d 0. For a xed b 0 let
2
Qb (d) = P (0, b, 0, d). Now, Qb (d) 0 if and only if 0 d b4 and hence by continuity
of Qb , Qb (b2 /4) must be zero. This implies that for all b 0, one variable polynomial
2
P (0, b, 0, b4 ) = 0, and hence this polynomial is always zero. This means that polynomial
2
x4 + bx2 + b4 has four real roots for all values of b . Contradiction!

6 . Let I and I be the incenters of triangles ABC and AT S respectively, and and
be the incircle of triangle ABC and circumcircle of T I S respectively.
We have
T DF = SDE = 90
A
1
EDF = EIF = 90
2
2
T DS = 360 90 90 (90
= 90 +

A
.
2

A
)
2

On the other hand we have T I S = 90 + A


2 ; therefore, T DI S is cyclic and D is on

. We know that, center of is the midpoint of T S of the circumcircle of triangle AT S ,


so it is on the line II . Hence two circles , intersect at D, and I is the intersection
of and the line joining the two centers. Thus I is inside .
24

25

Third Round
1 . a) In each acyclic orientation of G, there is at least one vertex (say vi ) such that all
edges connected to that are directed toward vi , because if not the orientation will have
a cycle which contradicts the fact that the orientation is acyclic. By omiting this vertex
and all edges connected to it, we obtain an acyclic orientation for the graph G vi .
The number of acyclic orientations of G vi is f (G vi ), but for some orientations of
G there exists more than one vertex with the property discussed. Therefore, the assertion
is proved.
b) Every acyclic orientation of G e results in at least one acyclic orientation of G
because if vi and vj are two endpoints of edge e, then if e cannot be oriented from vi
to vj , it means that there is a path (in G e) from vj to vi . Similarly, if e cannot be
oriented from vj to vi , it means that there is a path (in G e) from vi to vj .We conclude
that because of two directed paths from vi to vj and from vj to vi , there exists a directed
cycle containing vi and vj in G e, which is a contradiction.
So each acyclic orientation of G e results in one or two acyclic orientaions of G.
Now, if edge e can be oriented in two directions, then in G e there is no directed path
either form vj to vi or from vi to vj . So if we omit e to get G , as stated in the problems
statement, an acyclic orientation of G can be found, and the assertion is followed.
c) It suces to consider the complete graph with n vertices. In this case f (G) = n!,
and according to part (b) we have
f (G e) = n! (n 1)! = (n 1)(n 1)!.
Therefore
that

f (G)
f (Ge)

n!
(n1)(n1)!

n
n1 .

For each > 1, there is a natural number n such

n
1
=1+
<
n1
n1

and this completes the proof.

2 . The idea is to remove some neighborhoods of A and B , because near these points
AA
we cannot bound BB
. Let Z be the intersection of the ray AB with the boundary of S
and let ZB intersect AA in A . A is between A and A since S is convex. So, if we let
S be the set of those points X such that AA > 31 BB , we have S S . Hence, it is
enough to prove that the area of S is at least 6.
By Menelaus's theorem, we have
AA XB

A X B B
AA

A X

BZ
=1
ZA
XB
= 2.
BB

26

To omit of A X , we write

AA
A X

in terms of AA and AX and we treat B X similarly:

A X
1 XB
=

AA
2 BB
AX
1
BX
= 1 (1
)

AA
2
BB
AX
1 BX
1
=
+ .

AA
2 BB
2

1
BX
BX
Let D1 be the set of those points X such that 12 BB
+ 2 BB for some constant
(for suitable , D1 is a neighborhood of B ). If X D1 , then

AX
AA
AA

BB

BX
BB
1 AX

.
BX

<
>

AX
Let D2 be the set of those points X in the plane such that BX
<
is a neighborhood of A). If X is not in either of D1 or D2 , then

(for suitable , D2

AA
1
1
> =

BB
3
3
as desired. So

S D1 D2 S S .

Now, we calculate the areas of D1 and D2 . If < 3, then D2 is the interior part of the
Apolonius circle of A and B . Also,

BX
1
BX
1
1 BX

.
+

<

2 BB
2
BB
BB
2 1

X D1

So, if > 1 then D1 is a neighborhood of B similar to S . Now, we take = 32 . We


conclude that the area of D1 is ( 12 )2 times the area of S which is 10
4 = 2.5. Also, if C and
D are the intersection points of the boundary of D2 with the line AB and C is between
A and B , we have

AD
1
AD
1
=
= AD = 1
DB
2
AD + 1
2
AC
1
AC
1
1
=
= AC = .
CB
2
1 AC
2
3
So the diameter of D2 is 1 +

1
3

TX

4
3

and hence, the area of D1 is ( 32 )2 < 1.5. So

TSD1 D2
2TS TD1 TD2
> 10 2.5 1.5 = 6.

where TK represents the area of gure K in the plane. Hence, the assertion is proved.
27

. First we prove a lemma.

Let a > b be two positive integers such that


positive integer l such that b < (2l ) = 2l1 < 2l < a.
Lemma.

a
b

> 4.

Then there exist some

Proof of lemma. There exists some positive integer m such that 2m1 b < 2m . So

a > 4b 2m+1 and hence for l = m + 1 we have b < 2l1 < 2l < a, as claimed.

We construct the sequence in the problem inductively. The assertion for n = 1 is


obvious. Now, suppose that a1 < a2 < < an is a sequence such that (a1 ) > (a2 ) >
> (an ). Let pN be the greatest prime factor of ai 's (pj is the j th prime number),
1 i n. Since for every x with prime factors greater than pN , the greatest common
divisor of x and ai is 1 for all 1 i n, we deduce that the sequence a1 x < a2 x < <
an x also satises the condition of the problem (because (ai x) = (ai )(x)). Now, our
goal is to nd such x and some positive integer y such that y < a1 x and (y) > (a1 x).
First we claim that there is a positive integer x with prime factors greater that pN such
x
x
1)
> 4 or equivaletly, (x)
> 4 (a
that (aa11)(x)
a1 .
To prove this, let xm = pN +1 pN +2 pN +m (m N). We have

)
N
+m
N
+m (
N
+m
pi
1
1
xm
=
=
1+

.
(xm )
pi 1
pi 1
pi 1
i=N +1

The sum
such that

1
i=N +1 pi 1 >
(a1 )
xm
(xm ) > 4 a1 .

i=N +1

1
i=N +1 pi

i=N +1

diverges by a well-known fact, so we can nd m

We let x = xm . Now, since

lemma there is some positive integer l such that (a1 x)


y = 2l leads to the new sequence

a1 x
(a1 )(x) > 4, according to the
< (2l ) < 2l < a1 x. So putting

y < a1 x < a 2 x < < a n x


which has n + 1 terms and satises the problem's condition.

4 . Obviously, if we are able to nd the dierent bag among n bags with k times using
the balance, for less than n bags also k times using the balance is enough. Denote by
f (k) the maximum number of bags such that the dierent bag among them can be found
with at most k times using the balance. Our goal is to nd f (k) for all k N.
(i) f (1) = 14. If n = 14, for 1 i 14 we put i 1 coins from the ith bag on the
balance, and if the resulting weight is 910 k (91 = 1 + 2 + + 13) grams, then the
(k + 1)st bag is dierent, so f (1) 14. To prove the converse inequality, suppose
28

that we have a number of bags and we want to use the balance only once. Let ai
be the number of bags for which we take exactly i coins from them. We have

a1 + 2a2 + + mam 100. ()


Note that for i 0, ai 1 because if we take the same number of coins from two
dierent
bags, we cannot distinguish between them. Now, we want to maximize
m
a
.
i=0 i Since the coecient of ak in () equals k , it's best for a0 to be maximized,
then a1 should be maximized and so on. We conclude that the maximum value will
be 14. So f (1) 14 and consequently f (1) = 14.
(ii) f (2) = 60. To nd f (2), again () should be satised.
Also, for any integer i 0,
we must have ai 14. Here we want to maximize m
i=0 ai . The maximum value
will be gained when a0 = a1 = a2 = a3 = 14 and a4 = 4. Therefore f (2) = 60.
(iii) f (3) = 140. By a similar argument, here () should be satised, and also for each
positive integer i 0, we must have ai 60, so the maximum value will be gained
when a0 = a1 = 60 and a2 = 20; therefore, f (3) = 140.
(iv) Finally, if k >
3, then ai 100 and we must have a0 f (k 1), so we get the maximum value of m
i=0 ai when a0 = f (k1) and a1 = 100. Hence f (k) = f (k1)+100.
Upshot.

[1, 14]
1

[15, 60]
2

[61, 140]
3

[100k + 40, 100k + 140], k N


k+3

. Let
Q(x, y, z) = P (x, y, z) + P (y, x, z)
R(x, y, z) = P (x, y, z) P (y, x, z).

By these denitions it is obvious that Q is a symmetric polynomial and R is an


antisymmetric one. Note that R(x, x, z) = 0, so R is divisible by x y . Similarly, y z
and z x also divide R. Therefore there is a polynomial S such that

R(x, y, z) = (x y)(y z)(z x)S(x, y, z).


And it's obvious that S is a symmetric polnomial. We conclude that

1
1
P = Q + (x y)(y z)(z x)S.
2
2
Every symmetric polynomial in three variables can be written as a polynomial of elementary symmetric polynomials P1 = x + y + z , P2 = xy + yz + zx and P3 = xyz . If we
put P4 = (x y)(y z)(z x) (which obviously is cyclic) the proof is complete.
29

6 . We call a convex n-gon A1 A2 An with vertices from lattice points good if for each

integer 1 i n 1, vector Ai Ai+1 lies in the rst quadrant of the cartesian plane.
Throughout the solution all polygons are considered to have vertices from lattice points.
We denote the least possible value for the area of a convex n-gon and the area of a
good convex n-gon by f (n) and g(n), respectively. Furthermore, in such polygons let

ui = Ai Ai+1 (1 i n 1). We claim that g( n4 ) f (n) g(n). The rightmost


inequality is obvious, since good polygons are a subset of all such polygons. To prove the
leftmost inequality, note that for each convex n-gon P the rightmost, leftmost, topmost
and bottommost vertices divide the perimeter of the polygon into at most 4 parts, so at
least one part has at least n4 veritces and the area of the convexhull of these vertices is
at least g( n4 ) so the claim is proved.
Now, it suces to prove the assertions for g(n). (g(n) = O(n2 ), ...)

Lemma 1. The area of a good n-gon P with vectors u1 , u2 , . . . and un1 equals 21 i<j |ui
uj |.
Proof of lemma. Suppose P = A1 A2 An and A1 is the origin. Then the area of

triangle A1 Ai Ai+1 equals 12 |ui Ai | = 12 i1


j=1 |ui uj | (Note that all the cross products
have the same sign). The area of P equals sum of the area of these triangles, so by
summing these areas the lemma is proved.
Now let us return to the main problem:
a) Let ui = (1, i), Since the slope of vectors u1 , u2 , . . . and un1 are increasing, the
n-gon that they determine is good. Applying lemma 1 implies

S =

1
2

|ui uj | =
i<j
1 2
j
=
4
j

1
2

(j i)

i<j
n(n+1)(2n+1)
24

1
2

j(j1)
j

and this implies the assertion.


b) According to the lemma, the area of each good n-gon is at least
( )
|ui uj | is a positve integer, so g(n) n1
2 .

(n1)
2

, because

c) It suces to prove the following claim.


Claim 1.

For suciently large n, the area of each good (n + 2)-gon is at least

1 25
24 n .

By lemma 1, the following claim implies claim 1.


Claim 2.

we have:

For suciently large n and integer vectors u1 , u2 , . . . un+1 with distinct slopes

|ui uj |

i<j

30

1 5
n2 .
12

If

|ui uj | 16 n 2 for 1 i n + 1, summing these expressions implies the claim.

So without loss of generality we can assume that

1 3
|ui un+1 | n 2 .
6

We do not have the assumption of monotonocity of slopes


any further. For each i,
let Ci = {ul : |ul un+1 | = i} and ki = |Ci |. Obviously
ki = n. Elements of Ci lie
on two lines parallel to un+1 , so for each j n, at most 4 numbers among the values
|uj ul | (ul Cj ) can be equal (except number 0 when uj Ci which is unique).
Since these values are all integer, we have
( ki )

|uj ul | 4 4 .
2
ul Ci

Therefore,

|uj ul | 4

|uj ul |

1 2 n
ki ,
8
2
i

n 2 n2
ki ,
8
2
i

j,l

ln

and

( ki )

3
2

but
ki = n and
iki 16 n . Now, according to the following lemma we have
3
ki2 32 n 2 , and therefore for suciently large n

|uj ul |

j,l

1 5
3 5 n2
n2
n2
16
2
6

and this completes the proof of the claim.

Let k1 , k2
, . . . , kn be nonnegative real numbers such that

3
iki b. Show that
ki2 a4b .
Lemma 2.

ki = a

and

Proof of lemma. Suppose that values a and b are constants and k1 , k2 , . . . , kn are

variables satisfying the conditions for which


ki2 is minimum. We conclude that ki 's,
1 i n, are in decreasing order because
2 if ki < kj for some i < j then we can substitute
both of them with 21 (ki + kj ) and
ki will have a smaller value.
We claim that k1 , k2 , . . . , kn must be a block of an arithmetic progression unless they
k
+k
are zero. Suppose that kj1 , kj and kj+1 are nonzero and kj = j1 2 j+1 . If we change
these three to kj1 + x, kj 2x, kj+1 + x, it is easy to checkthat this new sequence also
satises the conditions. Thus for the dierence of the value
ki2 for these two sequences
we have
2
2
= ((kj1 + x)2 + (kj 2x)2 + (kj+1 + x)2 ) (kj1
+ kj2 + kj+1
)

31

= 6x2 + 2x(kj1 + kj + kj+1 ).


Note that the coecient of x is nonzero. Hence we can choose a small value of x
having a dierent sign from it's coecient for to be less than 0. Therefore, each
three consecutive nonzero numbers among ki 's must form an arithmetic progression. By
modifying n if necessary, we can assume that the sequence k1 , k2 , . . . and kn is a decreasing
arithmetic progression of nonzero numbers, so ki = r si for constants r, s 0. We have

a =
nr

i 2
c = r i s i b.
Hence

ki2 = nr2 2rs

i + s2

so

i2 = r(nr s

i) s(r

is

i2 ),

ki2 = ra sc.

The values of r and s can be calculated from the linear system of equations above in
terms of a, b and c:

a i nc
a i2 c i
, s=
r=

.
n i2 ( i)2
n i2 ( i)2
By

Cauchy-Schwarz Inequality we obtain that

ki2

( ki )2
a2

= .
n
n

Since r sc is negative we have

a i2 c i an i + n2 c 0
2
a( n (n+1)
n(n+1)(2n+1)
) c(n2 n(n+1)
)
2
6
2

a(n 2n+1
)

c
3

n 3 ac + 1 4 ac .

cn(n+1)
2

The last inequality follows from the fact that c a (because c =


Finally we have

ki2

iki

ki = a).

a2
a3
a3
a3
=

n
an
4c
4b

as desired.

Claim 3.

The area of such good (n+2)-gon is at least


32

1
3
200 n

and therefore f (n) = (n3 ).

To prove this claim it suces to prove that for each set of integer vectors u1 , u2 , . . .
and un+1 with distinct slopes, we have

|ui uj |

i<j

1 3
n .
100

1 2
|ui un+1 | 50
n . Ci and ki

are dened as before. In this part we must prove a better bound for ul Ci |uj ul |. If
ul , ul Ci lie on a line parallel to un+1 , then ul ul is an integer multiple of un+1 (we
can suppose un+1 is not an integer multiple of any other vector, if else we can change it
with a shorter vector pointing at the same direction). So uj ul uj ul is a multiple
of uj un+1 . Note that elements of Ci lie on two lines parallel to un+1 . Consequently

Similar to the previous claim, it's enough to assume

|uj ul | 4J

ul Ci

( ki )
4

where we have assumed uj CJ .


Summing these inequalities on index i implies

( ki )
4 .
|uj ul | 4J
2

ln

Then by summing on index j we have

|uj ul | 4(

iki )

j,l

Since
Hence

( ki )

ki = n and

j,l

iki

1 2
50 n ,

|uj ul | ( 18

=(

)
(
n
1
2
ki
.
iki )
8
2

by applying lemma 2 we obtain

ki2

25
2 n.

1
iki )( ki2 )
25 )(

1 1
4(8

1
ki ) 3
25 )(

1 3
50 n .

7 . We show that bridges satisfying the problem's conditions can be built no matter what
the conguration of the highways is.
Throughout the solution we denote the highways by C1 , C2 , . . . and Cn and by G the
graph determined by those highways (crossroads are vertices of the graph and highways
between two crossroads are edges of the graph).
Lemma. Faces of G can be colored by two colors in a way that the colors of every two
adjacent faces is dierent (Two faces are adjacent if they have an edge in common).

33

Proof of lemma. We proceed by induction on number of edges of graph G. We call a

coloring of faces of G satisfying the mentioned property, a good coloring. Suppose that
C is the shortest path in G that lies in just one highway and assume C C1 . Note that
because of minimality, C cannot cross itself, so is a simple curve, therefore dividing the
the plane into two regions. Now, C C1 is a closed or maybe an empty curve.
Suppose that H is the graph consisting of C C1 , C2 , . . . and Cn . Since the number of
edges of H is less than G, H has a good coloring according to the induction hypothesis.
Now, we add curve C to H and change the color of faces inside C . It is very easy to
check that this is a good coloring of G. The base case is when G is empty. In this case
we have only one region which obviously has a good coloring!

Finally consider a good coloring (by colors black and white) for the graph G and
an arbitrary orientation for each highway. We have two types of edges. For some edges
when we are moving through ine (in it's specied direction) our right face is white. We
call such edges, edges of type 1 and others, edges of type 2. Build a bridge in the way of
each edge of type 1.
Note that in each crossroad exactly two of the edges are directed into the crossroad
and only one of those two is of type 1. Hence our construction do not cause any contradictions.

8 . a) We claim that a set cannot exist with this property. Assume to the contrary that
there exists a set S satisfying the problems condition and let n N , n
/ S . Thus
exactly n elements of S are coprime to n. Therefore, there exist innitely many prime
numbers which do not belong to S . Let p and q be two of them. Since p
/ S exactly p
elements of S are not divisible by p (coprime to p), for each > 1, p S . This is true
because if p
/ S , there must be p elements of S coprime to p (and therefore coprime
to p), but we have shown that S has exactly p elements of this kind. Finally, the greatest
common divisor of p and q is 1, so innitely many elements of S are coprime to q
/ S,
a contradiction!
b) We construct set the S in the following way:
Suppose a1 , a2 , a3 , . . . are elements of S and at rst all ai 's, i N, are 1. In each step
we multiply ai by some primes.
For n N in step n:
(i) We multiply an by some new primes p2n1 and p2n so that an become greater than
an1 (Note that pn is the nth prime number).
(ii) Some elements among {a1 , a2 , . . . , an1 } are coprime to an . let bn be the number
of such elements, so bn < n < an . Let tn = an bn .
(iii) Because of the type of construction, innitely many number of ai 's, i > n, are
coprime to an . We leave rst tn and assume that the tn th number is am .
34

(iv) In the sequence am+1 , am+2 , . . . multiply the rst 2n1 terms by p2n1 , the next
2n1 terms by p2n and so on.
Therefore at the end of step n:
(i) Exactly an numbers are coprime to an .
(ii) For each sequence (q1 , q2 , . . . , qn ) of prime numbers such that qi {p2i1 , p2i }, there
exist innitely many numbers k such that ak has exactly these prime factors. So
for each ai there exist innitely many numbers k such that ak has prime factors
dierent from those of ai and consequently is copirme to ai .

35

Team Selection Test


1 . Let J and I be foot of the perpendicular lines from J and I to line BC , respectively. Furhtermore, throughout the solution we denote by p(XY Z) and S(XY Z) the
semiperimeter and area of triangle XY Z , respectively.
J is the tangency point of B -excircle of triangle ABH and line BH , so we have

BJ = p(ABH) = AB+BH+AH
. Also we have BP = AB+BC+CA
and hence P J =
2
2
AH+CACH

. II is the radius of C -excircle of triangle ACH , therefore


BJ BP =
2
II =

S(ACH)
AH.CH
=
.
p(ABH) AH
AC + CH AH

We claim that P J = II .

P J = II

AH.CH
AH + CA CH
=
2
AC + CH AH

2AH.CH = CA2 (CH AH)2 = CA2 CH 2 AH 2 + 2CH.AH


CA2 = CH 2 + AH 2 .
Which is true because of the Pythagorean Theorem. By a similar argument we get
P I = JJ . Therefore, the right-angled triangles P II and P JJ are congruent, so

IP I = P JJ = 90 JP J IP J = 90

HI and HJ are angle bisectors of AHB and AHC , respectively, and therefore
IHJ = 12 (AHB + AHC) = 90 . Since IP J = IHJ = 90 , we deduce that I , P ,
J and H lie on the same circle.
36

2 . We claim that the maximum number of such sets is 23 .


We proceed by induction on n. For the base case we must check the claim for n = 1, 2
which is obvious. Suppose that the claim is true for n. For n + 2 let G be the family
of those selected subsets of {1, 2, . . . , n + 2}. We divide G into four sets with respect to
their intersection with {n + 1, n + 2}.
n

G0 = {A G|n + 1
/ A, n + 2
/ A}
G1 = {A G|n + 1 A, n + 2
/ A}
G2 = {A G|n + 1 A, n + 2 A}
G3 = {A G|n + 1
/ A, n + 2 A}
For i = 0, 1, 2 let Fi be the family of A {n + 1, n + 2} for each A Gi . We claim that
for each B {1, 2, . . . , n}, B is in at most one of the sets F0 , F1 and F2 , Because if
B Fi , Fj , where i < j , it means that there exist Bi and Bj in G such that

Bi {n + 1, n + 2} = B, Bj {n + 1, n + 2} = B.
Now, Bi Bj but |Bj Bi | = |Bj | |Bi | = j i < 3 which is a contradiction, so
n
|G0 | + |G1 | + |G2 | 2n . The number of elements of G3 is at most 23 according to the
induction hypothesis, so
n
n+2
|G| 2n + 23 = 2 3 .
To construct an example for this upper bound, for i = 0, 1, 2 let Si be the family of
subsets A {1, 2, . . . , n} such that |A| i (mod 3). We have

|S0 | + |S1 | + |S2 | = 2n .


n

Since these sets are disjoint, one of them has at least 23 elements which obviously
satises the problem condition.

3 . Let Qk (x) =
we get

i=0 a(i, 2k

2i)xi . According to the recurrence relation for a(m, n),


Pk (x) = xPk1 (x) + Qk (x)

Qk (x) = xQk1 (x) + Pk1 (x).


So Qk (x) = Pk (x)xPk1 (x) and therefore, Pk (x)xPk1 (x) = x(Pk1 (x)xPk1 (x))+
Pk1 (x). Finally, we get

Pk (x) = (2x + 1)Pk1 (x) x2 Pk1 (x).


Hence we get a recurrence relation for Pk+1 (x) where k 2, P1 (x) = 3x + 1 and
P2 (x) = 5x2 + 5x + 1.
37

For each positive integer k 2 all of the roots of Pk (x) and Pk1 (x) are real
and distinct. Furthermore, if a1 < a2 < < ak1 and b1 < b2 < < bk are roots of
Pk1 and Pk , respectively, we have
Claim.

b1 < a1 < b2 < a2 < < ak1 < bk .


Proof. We proceed by induction. For the base case k = 2, 1
3 is the only root of P1 and

1
P2 ( 1
3 ) < 0 so 3 lies between the two roots of P2 (x).
Suppose that Pk1 (x) and Pk (x) satisfy the induction hypothesis. We know Pk+1 (x) =
(2x + 1)Pk (x) x2 Pk1 (x). Now, we consider the signs of Pk and Pk1 on dierent real
numbers. First suppose that k is even.

x
Pk
x2 Pk1

b1
0

a1

b2
0
+

a2
+
0

bk1
0

ak1

bk
0
+

+
+
+

Since Pk+1 (x) = (2x + 1)Pk (x) x2 Pk1 (x), for the sign of Pk+1 in respective bi 's we
have

x
Pk+1

b1
+

a1

b2

a2

bk1
+

ak1

bk

+
+

According to the change of signs of Pk+1 (x) and by the Mean Value Theorem, for
each 1 i k 1, Pk+1 has a root between bi and bi+1 . Also, it has one root less than
b1 and one root greater than bk , which completes the proof for even values of k . The
case where k is an odd number is similar, the only dierence being the signs of b1 and .
Note that
(i) 0 is not a root of any of the Pk 's. Because Pk (0) = a(0, 2k + 1) = 1, Pk1 (x) and
x2 Pk1 (x) have the same sign.
(ii) The coecient of xk in Pk (x) equals a(2k, 1) > 0, so Pk (+) > 0 and the sign of
Pk () is related to the parity of k .

. By considering centers of the hexagons, we get the following equivalent problem:

"Two vertices and of the triangular lattice are called equivalent if is equal
to sum of nitely many vectors from the set:

A = {(ni mj), (nk + mi), (nj + mk)}


Where i = (1, 0), j = ( 21 ,
coordinates.

3
2 )

and k = ( 12 ,
38

3
2 )

according to the normal Cartesian

What is the maximum number of nonequivalent vertices?"


Note that k = i + j , thus we can determine vectors of set A using only i and j :

A = {(ni mj), ((n + m)i + nj), (mi + (m + n)j)}.


Since the rst vector ni mj is the dierence of the two others, two vertices and
are equivalent i their dierence can be written as a linear combination of vectors
(n + m)i + nj and mi + (m + n)j with integer coecients.
Note that for integers a, b, x and y the space generated by the vectors {ai+bj, xi+yj}
(using integer coecients) equals the space generated by vectors {(a x)i + (b y)j, xi +
yj}. Therefore, we can change former vectors to get simpler vectors to work with. Note
that in this change the value of ay bx (determinant of the matrix formed by vectors
(a, b) and (x, y) as rows) is invariant, because ay bx = (a x)y (b y)x. Now,
in a similar way to Euclidean Algorithm, if a x we change {ai + bj, xi + yj} by
{(a x)i + (b y)j, xi + yj}. Repeating this process several times leads to vectors of
the form si + rj and 0i + tj . It means that we can suppose the coecient of i for one of
them is zero. hence, if we want to go from one vertex to another, the dierence of the
coecients of i must be a multiple of s, since tj does not aect the coecient of i. As a
result, for two equivalet vertices, we can rst use vector si + rj several times to match
their rst coordinate. Then, we must use tj to match the second coordinate. Hence we
have st nonequivalent vertices. But
[
]
s r
st = det
0 t
which is invariant during the process.
[
]
[
]
s r
m+n
m
det
= det
= m2 + mn + n2
0 t
n
m+n
So the maximum number of nonequivalent vertices is m2 + mn + n2 .

Let A 2 (mod 3) be a positive integer. Then there exists a prime numbr p


such that p 2 (mod 3) and p ||A where is an odd integer.

Lemma 1.

Proof of lemma. Assume to the contrary that there is not such a prime number p.

k
1 2
Therefore, if A = p
1 p2 pk is the prime factorization of A, we have two cases for
pi 's, 1 i k , to consider:

i
Case 1. pi 1 (mod 3) p
i 1 (mod 3).

39

2
i
Case 2. pi 2 (mod 3) and 2|i p
i (pi )

i
2

1 (mod 3).

As a result, we must have A 1 (mod 3) which is a contradiction.

Let p 2 (mod 3) be a prime number. Show that {03 , 13 , . . . , (p 1)3 } is a


complete residue system modulo p.

Lemma 2.

Proof of lemma. Obviously i3 03 (mod p) i i 0 (mod p). Suppose that p - i, j .

Our goal is to show that i3 j 3 (mod p) i i j (mod p). One part of the proof is
obvious. To prove the other part, suppose that p = 3t + 2. Then by Fermat's Little
Theorem, we have i3t+1 j 3t+1 1 (mod p). Hence, we have

i3t i i3t+1 j 3t+1 (j 3 )t j i3t j

(mod p).

Since (i, p) = 1, we get i j (mod p).


Now we are ready to solve the main problem. We claim that there is no such triple.
Assume to the contrary that

a2 + b2 + c2 = 2013k(ab + bc + ca)
for some positive integer k .
First, without loss of generality we can suppose that a, b and c have no common factor,
because if (a, b, c) = d > 1, we can divide them by d to get a new triple with no common
factor. We have (a + b + c)2 = (2013k + 2)(ab + bc + ca). 2013k + 2 2 (mod 3), so by
lemma 1 there is some prime number p 2 (mod 3) such that p2n+1 ||2013k + 2 (n 0).

p2n+1 ||2013k + 2 p2n+1 |(a + b + c)2 p2n+2 |(a + b + c)2


p2n+2 |(2013k + 2)(ab + bc + ca) p|ab + bc + ca.
As a result, p|a + b + c and p|ab + bc + ca. Hence

0 ab + bc + ca ab + c(a + b) ab + c(c)
c3 abc

(mod p) ab c2

(mod p)

(mod p).

By a similar argument, a3 b3 abc (mod p), so by lemma 2 we deduce a b c


(mod p) and since p|a+b+c and 3 - p, we nd that p divides a, b and c, which contradicts
our assumption that (a, b, c) = 1.
40

6 . Without loss of generality, we can suppose that 1 is shorter than 2 . Let l be the
common external tangent of 1 and 2 and let it intersects l at O. Let OA be tangent
to 1 at A, B := 1 1 , C := 1 l , D := l 2 , E := 2 2 and let OF be tangent
to 2 at F . We must prove that A, O and F are collinear.
Let the common internal tangent of 1 and 1 meet l at O , so
O X.O Y = O B 2 = O X .O Y .
Thus the power of O with respect to 2 and 2 is equal to this value. Hence the circle
C1 with center O and radius O B passes through E and is orthogonal to all arcs 1 , 2 ,
1 and 2 . We also have OA2 = OX OY = OD2 and OF 2 = OX OY = OC 2 , So
the circle C2 with center O and radius OA is orthogonal to both 1 and 2 and also the
circle C3 with center O and radius OF is orthogonal to both 1 and 2 .
We use the following classic lemma without proving it.

Let circle be orthogonal to circles C1 and C2 and intersect them in {A1 , B1 }


and {A2 , B2 }, respectively. Let O, O1 and O2 be the centers of , C1 and C2 , respectively.
Suppose that the right angles OA1 O1 and OA2 O2 have dierent orientations; i.e. one
of them is clockwise and the other one is not. Then the pairs {A, A } and {B, B } are
direct anti-homologous points and pairs {A, B } and {A , B} are inverse anti-homologous
points for C1 and C2 .

Lemma 1.

Using the lemma for circle C2 , we deduce that A and D are inverse anti-homologous
points for 1 and 2 (we do not check the conditions here for brevity). Using the lemma
again for circle C1 , we get that B and E are also inverse anti-homologous points for 1
and 2 . So, the quadrilateral ADEB is cyclic.
Similarly, by using the lemma twice for circles C1 and C3 , we get that pairs {B, E}
and {C, F } are inverse anti-homologous points for 1 and 2 . So the quadrilateral BEF C
is also cyclic.
According to the lemma for circle C1 , points B and E are direct anti-homologous
points for 1 and 2 , and so are C and D. Hence, the quadrilateral BCDE is also cyclic.
By the last three paragraphs, we deduce that all points A, B, . . . , F lie on the circumcircle of triangle BCD, which we call it C4 (checking distinctness of the points is not
mentioned here). Now, circles C2 , C3 and C4 have a line of symmetry, so their intersections and centers also have a line of symmetry. It follows that O, A and F are collinear
and the problem is solved.
Second Solution

lemma.

. To prove that A, B, . . . , F lie on a circle, we can use the following

If a cricle is tangent to circles C1 and C2 , then the points of tangency are


anti-homologous points for C1 and C2 . If none of C1 and C2 contains the other and is
tangent to them both internally or both externally, then the points of tangency are direct
anti-homologous points.


Lemma 2.

41

Note that these lemmas remain true if one of the circles is a line (considered as a
circle with innite radius).
If we suppose that interior of l (considered as a circle) is the half-plane not containing
X , then according to lemma 2 for circle 1 , points B and C are direct anti-homologous
for circles 1 and l . By applying lemma 1 for circle C2 , points A and D are also antihomologous points for 1 and l . Using the fact that the arcs are all in one side of l, it
is easily seen that A and D are also direct anti-homologous points, so ABCD is a cyclic
quadrilateral.
Similarly, by considering anti-homologous points for pairs of circles {2 , 1 }, {l , 2 }
we conclude that quadrilaterals BCDE and CDEF are also cyclic, so points A, B , C ,
D, E and F lie on a circle which we call it C4 .
The rest of the proof is the same as the rst solution.

. Dene points A, B, . . . , F and circles C1 , C2 and C3 similar to the rst


solution. We can suppose that X is between X and O, without loss of generality. Thus,
C1 and C2 are disjoint and C3 contains both of them. According to lemma 1 and the
fact that 1 lies entirely on one side of l, points A and B are direct anti-homologous
points for C2 and C1 . Similarly, B and C are inverse anti-homologous points for C1 and
C3 and points C and D are direct anti-homologous points for C3 and C2 . Pairs of points
{D, E} and {E, F } are also anti-homologous in a similar manner as are pairs {A, B} and
{B, C}, respectively.
Our motivation is that two anti-homologous points are related to each other by an
inversion depending only on the two circles. So the problem says that the composition of
six particular inversions is the identity. With this intuition in mind, we move all points
to C1 . Let A , C , D and F be points on C1 such that pairs {A, A } and {D, D } are

Third Solution

42

direct homologous points for circles C1 and C2 , and pairs {C, C } and {F, F } are inverse
homologous points for circles C1 and C3 , respectively.
We have
O D OD OC O C ,
so, A D passes through O . The lines A B and D E pass through the direct homothetic
center of C1 and C2 , which lies on l. The lines BC and EF pass through the inverse
homothetic center of C1 and C3 , which also lies on l. So, by Pascal's theorem for the
cyclic hexagon A BC D EF , we conclude that A F also passes through O . Hence, we
similarly have
OA O A O F OF,
so O, A and F are collinear and we are done.
Comment 1. If we extend the arcs to perfect circles, then the other external tangent

of 1 and 2 and the other external tangent of 2 and 1 also meet on l.

Comment 2. If we let 1 and 2 be in dierent sides of l, then the assertion is no longer

right. Instead, the corresponding internal common tangents meet on l. Also an assertion
like that of the rst comment holds.
Comment 3. We can solve the problem using hyperbolic geometry. If we let the half-

plane be the Poincar model, then the arcs and the common tangents are equidistant
curves; i.e. the locus of points with a constant distance from a given line. This solution
gives interesting generalizations of the problem. For example, see the following gures.
43

7 . First Solution. We use the notation A for matrices and Aij for the entry of row i
and column j .
First, observe that Aii pi , since the sum of the ith row is pi and the entries are
nonnegative. Similarly, Aii qi . Thus, Aii min(pi , qi ) and it follows that trace(A)
min(p1 , q1 ) + min(p2 , q2 ) + ... + min(pn , qn ). It remains to prove there is a matrix with
the property that Aii = min(pi , qi ) for every 1 i n.
We proceed by induction on n. The case n = 1 is obvious, since p1 = q1 . Assume
the assertion holds for n = 1, 2, . . . , k and consider the case n = k + 1. We can suppose
q1 p1 without loss of generality. Let A11 = q1 and Ai1 = 0 for 2 i n. We claim that
there exist nonnegative real numbers i := A1i for 2 i n such that these conditions
hold:
n

i = p1 q1 ,

i=2

i qi min(pi , qi ) for every 2 i n.

(1)

To verify our claim, rst we prove a lemma.

Let a, b1 , b2 , . . . , bk be nonnegative real numbers such that a b1 + b2 + + bk .


Then there exist nonnegative real numbers a1 , a2 , . . . , ak such that a1 + a2 + + ak = a
and ai bi for every 1 i k.
Lemma.

44

Proof of lemma. We use induction on k . For k = 1 the lemma is obvious. If bk > a,

we can set a1 = a2 = = ak1 = 0 and ak = a, so suppose that bk a. Let ak := bk .


Now, we must have a1 + a2 + + ak1 = a bk . but a bk b1 + b2 + + bk bk =
b1 + b2 + + bk1 and the assertion follows by the induction hypothesis.
We have
n

(qi min(pi , qi )) =

i=2

(qi min(pi , qi )) =

(pi min(pi , qi ))

i=1

i=1

p1 min(p1 , q1 ) = p1 q1 .
Hence, by the lemma we can nd nonnegative real numbers 2 , 3 , . . . , n such that
equation 1 holds. Let A1i = i for 2 i n. The remaining entries of A should satisfy

Ai2 + Ai3 + + Ain = pi for 2 i n,


A2i + A3i + + Ani = qi i for 1 i n.
We have qi i min(pi , qi ) by equation 1, so min(pi , qi i ) = min(pi , qi ). Hence,
the rest of the matrix can be lled using the induction hypothesis.
We can change the order of p1 , p2 , . . . , pn and q1 , q2 , . . . , qn arbitrarily
by changing corresponding rows and columns of the matrix. So we can suppose p1
q1 , p2 q2 , . . . , pk qk and pk+1 qk+1 , . . . , pn qn . Let Aii = min(pi , qi ) for 1
i n. Also let Aij = 0 if i = j and either i k or j > k . The remaining entries
{Aij : i > k, j k} should satisty:

Second Solution.

Ai1 + Ai2 + + Aik = pi qi for i > k ,


A(k+1)i + + Ani = qi pi for i k .
i.e. we should nd a matrix with nonnegative entries for the given row sums and column
sums. We can construct such a matrix using induction on the number of rows as follows.
By the lemma above, we can nd An1 , An2 , . . . , Ank such that their sum is pn qn
and Ani qi pi for each i k . Then, we delete the n'th row and replace qi pi by
qi pi Ani for each i k . We can proceed by using induction and the desired matrix
will be constructed.
Comment. This problem originally arises from probability theory. Suppose

i pi = 1
and we want to have two random variables X, Y such that P(X = i) = pi and P(Y =
i) = qi for 1 i n. The problem wants the maximum possible value of P(X = Y ) over
all possible joint distributions of X and Y .

45

8 . We denote the problem's condition by . Let d be the common dierence and a + d


be the initial term of this arithmetic proression, where a and d are integers such that
a + nd > 0 for each n N (Note that a may be negative but a + d, a + 2d, . . . are all
positvie). Hence, d is positive and d > a.
Putting k = N + m for nonnegative integer m in gives us

a1 a2 aN +m |aN +1 aN +2 a2N +m ,
so by omitting equal terms:

a1 a2 aN |ak+1 ak+2 ak+N


for each k N (m 0). We denote this relation by . Let n0 > N be a natural number
congruent to 1 modulo P = a1 a2 aN . Then, by we have:

P |(a + (n0 + 1)d)(a + (n0 + 2)d) (a + (n0 + N )d).


Since P |n0 + 1, we get

P |a(a + d) (a + (N 1)d).

Since P = (a + d)(a + 2d) (a + N d), we get a + N d|a. Now, if a > 0, a + N d a, so


N d 0. Because N > 1 this implies d = 0, so an = a for all natural numbers n. Any
constant sequence is an answer.
If a = 0, an = a + nd = nd, and this is another sequence satisfying the problem's
condition.
If a < 0, a + N d |a| = a, so N d 2a. Since N 2 and d > a we have
2a < N d 2a which is a contradiction.
Finally, an = nd for d > 0 and constant sequences, are the only sequences satisfying .

9 . f is an increasing function so f (x) + x is strictly increasing and therefore injective.


Let A = f (x) + 2g(x) + 3f (y) and B = f (y) + 2g(y) + 3f (x). We have
f (A) + A = 3(f (x) + f (y) + g(x) + g(y)),
f (B) + B = 3(f (y) + f (x) + g(y) + g(x)).
Since f (A) + A = f (B) + B and f (x) + x is injective, we deduce that A = B :

f (x) + 2g(x) + 3f (y) = f (y) + 2g(y) + 3f (x).


This implies that f (x) g(x) = f (y) g(y) for all x, y > 0. Thus, f (x) g(x) equals
some contant value c. Putting x = y in the second equation gives g() = 3x + c > c, so g
takes any value greater than c and consequently, f takes any value greater than 2c. By
substituting g(x) = f (x) c in the rst equation we get

f (3(f (x) + f (y)) 2c) = 3(f (x) + f (y)).


46

Therefore, f (x) = x 2c for large values of x. Thus, we can nd values of x and y such
that f (x) = x 2c and f (y) = y 2c. Therefore, from the second equation we have

g(f (x) + y + g(y)) = g(x 2c + y + y 3c) = x + 2y 8c


2x g(x) + f (y) + y = 2x x + 3c + y 2c + y = x + 2y + c.
Hence, c = 0 and f (x) = g(x) for all x R+ . Furthermore, we have f (x) = g(x) = x
for large values of x. Now, let y be suciently large such that f (y) = y and let x be an
arbitrary positive real number in the second equation.

g(f (x) + y + g(y)) = g(f (x) + 2y) = f (x) + 2y


2x g(x) + f (y) + y = 2x x + y + y = x + 2y.
Therefore, f (x) = x for all x R+ , so we get f (x) = g(x) = x for all x R+ and it's
easy to check that this is indeed a solution.

10 . Denote by G the graph in the problem. We proceed by induction on the number of


edges of the complement graph G.
The base case is when G is a complete graph.
Let v1 , v2 , vi and vj be some four vertices of the complete graph G. By the problem's
condition for the 4-cycle v1 vi vj v2 v1 we get
N (vi v1 ) N (vi v2 ) = N (vj v1 ) N (vj v2 )
Where by N (vv ) we mean the number assigned to the edge vv . Thus, for each i, the
number N (vi v1 ) N (vi v2 ) is constant. We denote by K this constant value. Now, let
ni be the number we want to assign to the vertex vi . We must have

n1 n2 = N (v1 vi ) N (v2 vi ) = K
n1 + n2 = N (v1 v2 ).
By solving this system of equations we get the numbers due to the vertices v1 and v2 .
Now, for each i let ni = N (vi v1 ) n1 . We claim that these numbers satisfy the assertion
for complete graphs. For this, we must show that for every i and j , ni + nj = N (vi vj ).
For j = 2 this is equivalent to the second relation in the above system.
For other values of j , consider 4-cycle v1 vi vj v2 v1 . we have

N (v1 v2 ) + N (vi vj ) = N (v1 vi ) + N (v2 vj ).


Thus,

n1 + n2 + N (vi vj ) = n1 + ni + nj + n2 N (vi vj ) = ni + nj .

This nishes the proof of the base case.


47

Suppose that G does not contain edge vi vj .


We claim that we can add edge vi vj and assign a number to it such that the assigned
number satises the problem's condition for G {vi vj } and therefore, according to the
induction hypothesis the statement holds for G {vi vj }.
If the edge vi vj is not included in any even walk, we can choose an arbitrary number
for N (vi vj ), and if the edge vi vj is included in some even walk, the assigned number is
determined uniquely.
Furthermore, if the edge vi vj is included in two even walks C1 and C2 , C1 C2 {vi vj }
is also an even walk which satises the problem's condition. Therefore, the assigned
numbers to the edge vi vj by C1 and C2 are equal.

11

. Let a = x2 , b = y2 and c = z 2 for a y z 0. We have


x2 y 2 + z 2 (y + z)2 x y + z,

so x, y and
z are sides of a triangle.

Set Ax = y 2 + z 2 yz , Ay = z 2 + x2 zx and Az = x2 + y 2 xy . Then,

Ax Ay y 2 + z 2 yz x2 + z 2 xz (x y)(x + y z) 0
Ay Az x2 + z 2 xz x2 + y 2 xy (y z)(y + z x) 0.
Hence, Ax Ay Az .
Therefore,

xAz + yAy yAz + xAy (Az Ay )(x y) 0


yAy + zAx zAy + yAx (Ay Ax )(y z) 0.
For the main problem we have

2(xAz + yAy + zAx ) = (xAz + yAy ) + (yAy + zAx ) + xAz + zAx


(yAz + xAy ) + (zAy + yAx ) + xAz + zAx = (y + z)Ax + (x + z)Ay + (x + y)Az .
By

Cauchy-Schwarz Inequality we have


(x + y)Az = (x + y)

x2 + y 2 xy =

(x + y)(x3 + y 3 ) x2 + y 2 ,

and similar inequalities hold for (x + z)Ay and (y + z)Ax . Finally, summing these three
inequlities implies the assersion.

12

Let ABD be a triangle with circumcircle . Suppose that is a circle tangent


to sides AB and AD at E and F , respectively, and tangent to at T . M and N are

midpoints of shorter arcs AB and AD, respectively. Then tangents to at A and T , and
line M N are concurrent.

Lemma.

48

Proof of lemma.

1
1
N AF = N D = N A = AT N,
2
2

NA
F
2
so triangles T AN and AF N are similar, so N
N A = N T , and consequently N A = N F.N T .
In a similar way we can show that M A2 = M E.M T . Thus,

MA
AN

)2
=

M E.M T
. ()
N F.N T

T is the external homethetic center of and . This homothety maps E and F to M


E
MT
and N , respectively, so EF ||M N and by Thales' Theorem M
M F = N T . This and ()
imply
)
(
)
(
MT 2
MA
MT
MA 2 ME MT
=
.
=

=
.
NA
NF NT
NT
NA
NT
Therefore, M is the harmonic conjugate of N with respect to A and T , and hence tangents
to at A and T meet on the line M N .

We keep using the notations introduced in the lemma. We showed that K lies on the
line M N and now we want to show that K lies on the line I1 I2 . We have

KM
AM. sin(M AK)
AM sin(M CA)
=
=
.
=
KN
AN. sin(N AK)
AN sin(N CA)
49

M I2 sin(M CA)
r1
M I2 CI1
.
.
=
.
.
N I1
r2
sin(N CA)
N I1 CI2
As a result,

M K N I1 CI2
KN . I1 C . I2 M

= 1 and by

Menelaus' Theorem, K , I1 and I2 are collinear.

13 . Let A1 , B1 and C1 be reections of P with respect to BC , AC and AB , respectively. Moreover,


we denote by A2 and A the midpoints of B1 C1
and P A1 , respectively. Points B2 , C2 , B and C
are dened similarly. Note that A2 C P B1 and
A2 B P C1 , so A2 C AC and A2 B AB .
Therefore, A2 is the orthocenter of triangle AB C .

AB C AB P = 90 .
Similarly, AC B 90 . Furthermore, B AC = BAC < 90 because triangle
ABC is acute. Therefore, triangle AB C is acute, so its orthocenter A2 lies inside it
and therefore, inside triangle ABC . By similar arguments points B2 and C2 lie inside
trianlgle ABC . Hence, the centroid of triangle A2 B2 C2 lies inside triangle ABC , but the
centroid of triangle A2 B2 C2 coincides with that of triangle A1 B1 C1 , and this completes
the proof.

14 . First suppose that all rectangles have sides parallel to coordinate axis. There are
four types of right angles: p, q, x and y.
Suppose that there are a number of angles p and b number of angles y. Since each
rectangle is determined by these two angles, we get ab n, and by AM GM inequality
)2
(

we get a+b
ab n, so a + b 2 n. Similarly, if there are c number of angles q
2

and d number of angles x, we can deduce that c + d 2 n. Thus we have at least 4 n


right angles.
If there are dierent directions for rectangles,
we denote by n1 , n2 , . . . and nk the number
of rectangles in each direction. Since n = i ni , we get

4 n 4 n1 + 4 n2 + + 4 nk Number of angles.

15 . a) It suces to nd a sequence an such that a1 - an for all natural numbers n > 1 and
i|j if and only if ai |aj for all natural numbers i, j 2. Since (2m 1, 2n 1) = 2(m,n) 1,
the sequence dened by a1 = 2 and an = 2n 1 for n > 1 has this property.
50

b) We prove a stronger assertion.


For each function f : N N such that f (n) n for n > M , where M, n N, there
exists a sequence a1 < a2 < a3 < such that an is divisible by exactly f (n) terms of
the sequence for suciently large positive integers n.
Let a1 , a2 , . . . , aM be a strictly increasing sequence of prime numbers. Then, for k > M ,
set ak = [a1 , a2 , . . . , af (k)1 ]qk where qk is a new prime number specically for ak . Obviously, we can choose qk large enough so that ak became greater than ak1 .
By this construction ak is divisible by a1 , a2 , . . . , af (k)1 and itself which are f (k) numbers. Furthermore, ak is not divisible by aj for some other j , because each aj has its
specic prime number qj which has not appeared in the denition of ak .

16 . Let A = {f (x) f (y)|x, y Z}. By the problem's condition we know 3 A, and


our goal is to show that 1 A.
Obviously, A is closed under multiplication by 1 ((f (x) f (y)) = f (y) f (x)). We
claim that A is closed under summation.
Suppose that , A, so there are some integers m1 , m2 , n1 and n2 such that =
f (m1 ) f (m2 ) and = f (n1 ) f (n2 ). We have
f (m1 ) + f (n1 ) + f (f (m21 + n21 )) = 1
f (m2 ) + f (n2 ) + f (f (m22 + n22 )) = 1.
Subtracting these two equalities gives

+ = f (f (m22 + n22 )) f (f (m21 + n21 )) A


It is a well-known fact that any subset of Z closed under summation and multiplication
by 1 is kZ for some positive integer k . Since 3 A, k|3, so k = 1 or k = 3. If k = 3,
then for each x, y Z, f (x) f (y) is divisible by 3 and thus all numbers in the form f (x)
have the same remainder modulo 3. We denote this common remainder by r. Hence,

1 f (m) + f (n) + f (f (m2 + n2 )) r + r + r 3r 0

(mod 3),

which is a contradiction. Therefore, k = 1, so A = Z. Thus, 1 A, so there exist integers


c and d such that f (c) f (d) = 1.

17 . Let M , N and P be the midpoints of segments AB , AC and AD, respectively. Since


M and N are the centers of circles with diameters AB and AC , respectively, and AD is
the bisector of BAC , we have
MP
BD
AB
AM
MX
=
=
=
=
.
NP
CD
AC
AN
NY
Hence, P XM + P Y N = 180 , or P XM = P Y N . However, P XM < BXA =
90 , and P Y N < AY C = 90 ; therefore, the rst case cannot hold, so P XM =
51

P Y N (1). Furthermore, M P X = N P Y (2). By (1) and (2) we deduce that


triangles P XM and P Y N are similar, So
P M X = P N Y 180 B + 180 2ABX = C + 2ACY
ABX + ACY = 180 B C + 90 ABX + 90 ACY
ABX + ACY = BAC + XAB + CAY

(3).

Since AHB = AHC = 90 , H lies on the circumcircles of triangles AXB and AY C ,


so ABX = AHX (4) and ACY = AHY (5). Thus,

XHY = XHA + AHY = ABX + ACY = Y AX

By 3, 4, 5.

Since XY is the perpendicular bisector of segment AD, we get XAY = XDY .


Therefore, Y HX = Y DX , so the quadrilateral XHDY is cyclic.

18 . Suppose that the room in the problem is the square [0, n] [0, n] in the Cartesian
plane. Furthermore, throughout the solution we use these denitions:
A lattice point: A point in the plane with integer coordinates.
A lattice square: A square with lattice points as vertices and side lengths one.
A lattice edge: A side of a lattice square.
A lattice triangle: A right isosceles triangle of side lenght 1 and lattice points as
vertices.
52

A small triangle: A right isosceles triangle of side lenght


its hypotenuse.

2
2

and a lattice edge as

The boundary of each tile contains two lattice edges and two diameters of two lattice
squares, so we have two types of tiles. By these denitions the following lemma is obvious.

Suppose that e is a lattice edge of the lattice square S . Furthermore, assume


that e is not included in the boundary of any tile that lies in the same side of e as S . By
these conditions, there exist a lattice triangle and a small triangle containing e which is
contained in S that are not covered by any tiles.

Lemma.

Consider a graph with lattice edges as it's vertices. Two vertices are connected if and
only if their corresponding edges are on the boundary of the same tile. In this graph the
degree of each vertex is at most two. Furthermore, two connected vertices correspond to
parallel edges, and if the degree of a vertex corresponding to a horizontal (vertical) edge
is 2, one of the edges connected to this edge in the graph is above (right of) it and the
other lies below (left of) it. Hence, each connected component of the graph is a path
containing parallel lattice edges, and the order of vertices in this path coincides with
the order of one of the coordinates of the corresponding edges (x-coordinate for vertical
edges and y -coordinate for horizontal edges).
Thus, lattice edges on the line y = n lie on dierent paths. We denote these paths by
U1 , U2 , . . . , Un . Note that some of Ui 's might contain only one vertex. The bottommost
lattice edges of these paths are dierent. Let SUi be the lattice square below the bottommost edge of Ui . If SUi lies above the line y = 0, according to the lemma one of the
lattice triangles in SUi is not covered. We denote this triangle by T Ui . Furthermore,
the upper small triangle in SUi is not covered. We denote this small triangle by QUi . In
the same manner, we denote by D1 , D2 , . . . and Dn the paths containing lattice edges on
the line y = 0, by L1 , L2 , . . . and Ln the paths containing the edges on the line x = 0,
and nally by R1 , R2 , . . . and Rn the paths containing lattice edges on the line x = n.
Triangles T Di , T Li , T Ri , QDi , QLi and QRi are dened similarly. We have three cases:
Case1. Suppose that there exists a path like P containing one lattice edge of the lines
x = 0 and x = n. In this case all the tiles of paths U1 , U2 , . . . and Un are above
the tiles of the path P and tiles of the paths D1 , D2 , . . . and Dn lie below the
tiles of the path P . Now, 2n paths D1 , D2 , . . . and Dn , and U1 , U2 , . . . and Un
are all distinct, so the triangles T U1 , T U2 , . . . and T Un , and T D1 , T D2 , . . . and
T Dn can be dened and are disjoint. Since these triangles are not covered, and
the area of each of them is 12 , the sum of the area of these triangles is n. Hence,
the assertion is proved in this case.
Case2. There exists a path P containing one lattice edge of the lines y = 0 and y = n.
This case is similar to case 1.
53

An example for case 2.


Case3. There are not any paths of cases 1 or 2, so small triangles QDi , QUi , QLi and
QRi , for 1 i n, are all dened and are disjoint. Since these 4n triangles are
not covered by any tiles and the area of each is 41 , the sum of the areas of them
is n, so the proof is complete.

An example for case 3.

54

The Holy Month of Ramadan


and
Muslim Mathematicians

This year the IMO and the month of Ramadan are coincident.The holy month of
Ramadan, which is called as the month of mercy and forgiveness by Allah, is the ninth
month of the lunar months. According to the Islamic regulations, Muslims should fast
every day of this month from sunrise to sunset and refrain from eating and drinking,
except for those who are ill or traveling.

O, believers! Prescribed to you is the fast, as it was prescribed to those


before you, that you may guard against evil. (2-183)
For a certain number of days (you have to fast) but if any one of you be
sick, or on a journey, then (he shall fast a like) number of other days,
and those who are not able to do it, (for being too old, or permanently
sick) can be redeemed by feeding a poor. He that does good of his own
accord, it is better for him. And it is better for you to fast, if you could
knew it. (2-184)
The base of the lunar months is the rotation of the moon around the Earth. Each
lunar month begins when the new moon is sighted. Every twelve lunar months forms
a lunar year, which is almost equal to 354 or 355 days. Islamic regulations like the
law of fasting and also the important Islamic ceremonies like the Hadj and the religious
fetes are based on the lunar months, which explains why determining the lunar month
duration has been so important among the Muslims. The need for sighting the moon
crescent along with the need for orientation specially for determining the direction of
Kiblah (which is the Ka'ba in the city of Mecca) has caused the progress of Astronomy
in the Islamic countries.

The prediction of the visibility of the moon crescent has been one of
the most serious problems Muslims have faced, because the moon in its
rotation around the Earth is located near the sun from the view point
of an observer on Earth, at the beginning and the ending of the lunar
month. Therefore the crescent sighting is only possible in a short period
of time near sunset. Important factors in sighting the crescent are the
location of the moon and the sun relative to the horizon of the observer,
the pause of the moon after the sunset, the location of the moon in
its orbit relative to the Earth, the brightness of the crescent and the
weather. Muslim astronomers knew that determining the probability of
sighting the crescent requires solving complicated spherical astronomy
problems, so they could obtain the relative positions of the sun and the
moon on the horizon of the observer. The method used was in a way
with which the observer, using his calculations, knew the location whe55

re he was meant to search for the thin and dim crescent; therefore
observers tried to sight the moon crescent in the sunset of the 29th day
of the lunarmonth in a serene and an unhindered horizon, and they
would repeat their sightings the night after in case their sightings were
unsuccessful. One of the simplest criteria in sighting the crescent has
been brought up by Kharazmi (Mohammad ibn M
us a al-Khw arizm ),
the Iranian mathematician and astronomer. The basis of this criterion
was the calculation of the time dierence between the moonset and the
sunset at the local horizon. If this dierence reaches 12 degrees (equal
to 48 minutes of time) the moon crescent will be visible. As time passed,
Muslim astronomers obtained more complicated and precise criteria for
the prediction of the visibility of the crescent.
Translation Resource: Astronomy in Iran and the World of Islam (In
Persian), Ali Akbar Velayati, pp 237-241.

56

The Holy Month of Ramadan and Muslim


Mathematicians
The holy month of Ramadan, which is called as the month of mercy and
forgiveness by Allah, is the ninth month of the lunar months. According
to the Islamic regulations, Muslims should fast every day of this month
from sunrise to sunset and refrain from eating and drinking, except for
those who are ill or traveling.
O, believers! Prescribed to you is the fast, as it was prescribed to those
before you, that you may guard against evil. (2-183)
for a certain number of days (you have to fast) but if any one of you be sick,
or on a journey, then (he shall fast a like) number of other days, and those
who are not able to do it, (for being too old, or permanently sick) can be
redeemed by feeding a poor. He that does good of his own accord, it is
better for him. And it is better for you to fast, if you could knew it. (2-184)

The base of the lunar months is the rotation of the moon around the
Earth. Each lunar month begins when the new moon is sighted. Every
twelve lunar months forms a lunar year, which is almost equal to 354 or
355 days. Islamic regulations like the law of fasting and also the important
Islamic ceremonies like the Hadj and the religious fetes are based on the
lunar months, which explains why determining the lunar month duration
has been so important among the Muslims. The need for sighting the
moon crescent along with the need for orientation specially for
determining the direction of Kiblah (which is the Kaba in the city of
Makah) has caused the progress of Astronomy in the Islamic countries.
Read more about the role of Muslim astronomers and mathematicians in finding and using the
lunar months inside the booklet on pages 55 and 56.

You might also like